... am having trouble understanding JY's reasoning behind choosing correct ... necessary assumption question and PT34.2.2 is a sufficient assumption ... .
In PT34.2.2, the correct answer was (E ... , "do"?
In PT34.2.2, JY draws a conditional diagram ...